Questions tagged [probability-theory]

For questions solely about the modern theoretical footing for probability, for example, probability spaces, random variables, law of large numbers, and central limit theorems. Use [tag:probability] instead for specific problems and explicit computations. Use [tag:probability-distributions] for specific distribution functions, and consider [tag:stochastic-processes] when appropriate.

Filter by
Sorted by
Tagged with
0 votes
1 answer
26 views

Large deviation principle continues to hold when rate function is replaced by its lower semicontinuous regularization

Consider Lemma Suppose $I$ is a function such that (2.1) holds for all measurable sets A. Then (2.1) continues to hold if $I$ is replaced by $I_{\text{lsc}}$ proof: $I_{lsc}\le I$ and the upper bound ...
some_math_guy's user avatar
1 vote
1 answer
102 views

Intuition behind measurability of Markov kernels

Let $(X,\mathcal{A})$ and $(Y,\mathcal{B})$ be measurable spaces. In the definition of Markov kernel $K:X\times\mathcal{B}\rightarrow[0,1]$, it is required that $K(x,\cdot)$ is a probability measure ...
nexolute's user avatar
  • 113
3 votes
0 answers
55 views

General proof tactic for showing a.s. limit equalities?

I am interested in problems of the form: Show that $$\lim_{n\to \infty} X_n=C \text{ a.s.} $$ where $C$ is some known constant. I would like to know if there are any "classic" ways of ...
uniform_on_compacts's user avatar
4 votes
1 answer
167 views

Show a convergence in probability using Markov inequality

Let $(X)_{j\in \mathbb Z}$ be a discrete random process such that $$X_{j}= \theta X_{j-1}+ \epsilon_j, \quad (\epsilon_j) \overset{iid}{\sim} N(0,1), |\theta|<1 $$ How to show that $$P\left(\max_{1\...
André Goulart's user avatar
1 vote
1 answer
46 views

Total Correlation is difference of relative entropies in general?

Motivation Consider a finite set $[q]=\{1,\dots,q\}$, random variables $X_1,\dots,X_k\in[q]$, and their product $X=X_1\otimes\cdots\otimes X_k\in[q]^k$, i.e. the components of $X$ are independent. Let ...
Matija's user avatar
  • 3,558
0 votes
0 answers
51 views

Probability Notations and Continuity

I started working on probability theory and there is something I am confused with. Consider a transition function p on a countable and compact space $X$ that is $p(x^\prime|x)$. How can I understand ...
potfire's user avatar
0 votes
0 answers
78 views

Double Summation over a Subset of a Cartesian Product

From the "Probability & Statistical Inference, 9th edition" by Hogg, Tannis, Zimmerman, it is stated that one of the properties of the Joint Probability Mass Function of Random Variables ...
AdamLee123's user avatar
0 votes
0 answers
53 views

A simple example to practice justifying the interchange of derivative and expectation

I have been self-studying probability and measure theory. The condition I have for the interchange of derivative and integral is as follows, and comes from Folland, G. B. (1999). Real analysis: ...
ashman's user avatar
  • 952
2 votes
1 answer
86 views

Prove posterior mean is positive if and only if signal is positive, assuming zero prior mean [closed]

Suppose that: $\Delta \sim G$, where $G$ is a distribution that is symmetric about the origin I get a normally-distributed signal: $\hat{\delta} \: |\Delta, \tau^2 \sim N(\Delta, \tau^2)$ How can I ...
ryankessler's user avatar
0 votes
0 answers
99 views

Symmetric $\alpha$-stable distributions

Consider the expression appearing in the Levy-Kchinchine formula: $$\varphi(t) = e^{ita - \sigma^2/(2t)+\int_{-\infty}^\infty [e^{itx} - 1 -itx \mathbf{1}_{|x|<1}]\,{\rm d}\nu(x)},$$ but with $a =0,...
Robert Barg's user avatar
1 vote
1 answer
35 views

Limit of an integral related to characteristic functions

Suppose $X$ is a random variable with CDF $F_X$ and characteristic function $\varphi_X(t) = Ee^{itX}$. The inversion formula for the characteristic function gives: $$ F_X(b)-F_X(a)=\frac{1}{2 \pi} \...
LostStatistician18's user avatar
2 votes
0 answers
36 views

Reducing integrals over abstract spaces to integrals on $\mathbb{R}_{+}$ with respect to the Lebesgue measure

I am studying for a measure-theoretic based course in probability theory and I am stuck on a theorem that goes as follows. Definition First, it defines a measurable space $(E,\mathcal{E})$ to be ...
Fran712's user avatar
  • 373
1 vote
1 answer
86 views

How can we rigorously show conditional independence here?

Let $(E,\mathcal E,\lambda)$ be a measure space; $p:E\to[0,\infty)$ be $\mathcal E$-measurable with $$c:=\int p\:{\rm d}\lambda\in(0,\infty)$$ and $\mu$ denote the measure with density $\frac pc$ ...
0xbadf00d's user avatar
  • 13.4k
6 votes
1 answer
235 views

Categorical probability theory: Fritz' diagram categories

1. Context. In the seventh chapter of his 2019 paper A synthetic approach to categorical probability theory Tobias Fritz introduces certain diagram categories. Supposedly, these allow for a ...
Max Demirdilek's user avatar
0 votes
1 answer
32 views

Proving function of a martingale is integrable and measurable [closed]

Let $(X_n)_{n\in \mathbb{N_0}}$ be a martingale with values in $\mathbb{Z}$ such that $|X_n - X_{n-1}| = 1$ and $f: \mathbb{Z} \rightarrow \mathbb{R}$ an arbitrary map. I need to show that $f$ is ...
user996159's user avatar
0 votes
1 answer
43 views

Proving Hoeffeding's lemma [closed]

For $X$ a real-valued random variable such that $|X| \leq 1$ a.s., I need to prove the following by using the Jensen's inequality:$$\mathbb{E}[X] = 0 \implies \mathbb{E}[e^{\lambda x}] \leq \cosh(\...
user996159's user avatar
0 votes
1 answer
46 views

Prove the existance of a random variable satisfying a given property [closed]

We are given a real-valued rv $X$ with $|X|\leq 1$ a.s. I need to show that there exists a rv $Y$ with values in $\{-1,+1\}$ such that $\mathbb{E} [Y|\sigma(X)] = X.$ I do not know how the conditional ...
user996159's user avatar
0 votes
1 answer
40 views

Generalization error of a simple perceptron for a student-teacher network

I have been asked to prove the following expression given the following density probability function for a student-teacher perceptron network \begin{equation} P(x,y) = \frac{1}{2\pi\sqrt{Q-R^2}} \cdot ...
yuttokb's user avatar
  • 13
3 votes
2 answers
35 views

Show that the probability: $P(A^c \cup C) \geq 7/8$

Let $A,B,C$ be pairwise independent events with $P(A \cap B) = 0.1$ and $ P(B\cap C)=0.2$. Show that $P(A^c \cup C) \geq 7/8$. This is what I could do Since $P(B) \geq 2/10$ and $P(A)P(B)=1/10$, $P(A) ...
Euclidean_Space's user avatar
0 votes
0 answers
59 views

Arrival times of trains

Let $N^O$ and $N^U$ be an independent pair Poisson processes with intensities $\alpha_O$ and $\alpha_U$. We will model the incoming trains in the central station (trains that are arriving above the ...
Landy's user avatar
  • 35
4 votes
1 answer
81 views

Law of iterated logarithm and hitting time distribution

Let $X_i$ be a collection of independent standard normal variables and $S_n = \sum_1^n X_i$. The by the law of iterated logarithm $${S_n \over \sqrt{2n \log \log n}} > 0.8$$ infinitely often, where ...
Jyotirmoy Bhattacharya's user avatar
0 votes
0 answers
30 views

log-likelihood: changing relative contribution of two summation terms

Suppose I have a log-likelihood of the form $$\mathcal{L} = \sum_{i = 1}^{n} a_i + \sum_{j = 1}^{m} b_j,$$ where $a_i$ and $b_j$ are some independent log-probabilities. The problem is, the second sum $...
Dr. Timofey Prodanov's user avatar
1 vote
2 answers
137 views

Calculate $E[X|X+Y=z]$ for independent and identically distributed $X$ and $Y$.

Let $X,Y$ be independent and indentically distributed random variables taking values in $(0,\infty)$ and $z>0$. I want to calculate $E[X\mid X+Y=z]$. My intuition tells me, that its value should be ...
Vincent M. R.'s user avatar
0 votes
1 answer
120 views

Expected value of the reciprocal of sample standard deviation

Hopefully I am posting this in the right place, I've never used StackExchange before. The question I am answering is: Assume that X1, . . . , Xn are iid from $N(µ, σ^2)$ with unknown µ and $σ^2$. Find ...
jskarmeas's user avatar
0 votes
0 answers
35 views

Ash Probability and Measure 1.2.6 Proof Check

I was hoping someone would check my proof. The question is Let $f: \Omega \to \Omega'$ and let C be a class of subsets of $\Omega'$ Show that $\sigma(f^{-1}(C))=f^{-1}(\sigma(C))$ My proof is below. ...
Darst King's user avatar
3 votes
1 answer
96 views

The Sum of Independent Poisson Random Variables is Poisson: Converse

Suppose we have random variables $X \sim \text{Poisson}(\lambda)$ and $Y \sim \text{Poisson}(\mu)$ such that $X+ Y \sim \text{Poisson}(\lambda + \mu)$. Must $X$ and $Y$ be independent? Contrasting ...
Julius's user avatar
  • 1,321
2 votes
0 answers
183 views

The optimal constant for Burkholder-Davis-Gundy inequality

I have searched for some posts on math stack exchange, but I do not find a clear answer, can I have an explicit optimal constant for the BDG inequality $$\mathbb{E}\left[\int^T_0 \big|X(s)\big|^2ds\...
mnmn1993's user avatar
  • 445
3 votes
1 answer
275 views

Boundedness of $\dfrac{W_2(\mu_1+\varepsilon (\mu_2-\mu_1),\mu_1)}{\varepsilon}$ for 2 -Wasserstein metric

Let $\mathcal{P}_2(\mathbb{R}^{n})$ the space of Borel probability measures of finite second moment in $\mathbb{R}^{n}$ equipped with the $2$-Wasserstein metric $W_2$. Let $\mu_1$, $\mu_2 \in \mathcal{...
mnmn1993's user avatar
  • 445
2 votes
1 answer
115 views

Existence proof of conditional expectation

I am self-learning introductory stochastic calculus from the text A first course in Stochastic Calculus, by Louis Pierre Arguin. I'm struggling to understand a particular step in the proof, and I ...
Quasar's user avatar
  • 5,420
2 votes
1 answer
96 views

Understanding the derivation of the Poisson process. [closed]

I am learning about the Poisson Process and at the beginning of the proof, the following statement is left as an exercise for the reader: Let $N_{(a,b]}$ denote the number of random events in the ...
Newbie1000's user avatar
1 vote
1 answer
99 views

Simulating a continuous-time jump process

I've got a continuous-jump process $(X_t)_{t\ge0}$ with generator $$(Af)(x)=\tilde\lambda(x)((\tilde\kappa f)(x)-f(x)),$$ where $$\tilde\lambda=\lambda+c$$ for some bounded measurable nonnegative $\...
0xbadf00d's user avatar
  • 13.4k
2 votes
1 answer
180 views

Choose a random number btwn 0 and 1. Keep doing it until the sum of the numbers exceeds 1. Expected value of this sum?

This is a modified version to a pretty common problem; here's that classic problem: Choose a random number between $0$ and $1$ and record its value. Keep doing it until the sum of the numbers exceeds $...
APerson's user avatar
  • 85
0 votes
0 answers
50 views

Time Series Analysis and Recurrence Relations/Differential Equations

I am beginning to watch a video playlist on the subject of time series analysis, and it seems pretty clear both from notation and some of the terminology (such as "characteristic equation") ...
user10478's user avatar
  • 1,904
2 votes
1 answer
55 views

$\lim\limits_{n \to \infty}\frac{1}{n^2}\sum_{i=1}^n \mathrm{Var}[X_i] = 0$. Then $\frac{1}{n}\sum_{i=1}^n (X_i - \mathsf{E} [X_i])\to0$.

Let $(\Omega,\mathcal{F},P)$ be a probability space. Let $(X_k)_k$ be a sequence of independent random variable with finite second moment. If $\lim\limits_{n \to \infty}\frac{1}{n^2}\sum_{i = 1}^n \...
Klomanad's user avatar
  • 115
6 votes
0 answers
101 views

Show $\bigcup\limits_{n\mid n\geq N}A_{n,m}=\left\{\sup\limits_{n\mid n\geq N}\left|\frac{S_n(\omega)}{n}-p\right|\geq\frac{1}{m}\right\}$

Let be $m,n\in\mathbb{N}$, $p\in(0,1)$, $S_n$ is a binomially distributed random variable and $ \begin{align*} &A_{n,m}:=\left\{\omega\in\Omega\mid \left|\frac{S_n(\omega)}{n}-p\right|\geq \frac{1}...
Philipp's user avatar
  • 4,493
0 votes
0 answers
56 views

Dirac delta distribution and supremum

I have asked the same question here. My doubt follows from this paper pg. 546. The context is they find the lagrangian function and the primal variable is the set of all density functions. To make ...
Cherryblossoms's user avatar
2 votes
0 answers
48 views

Does "$n$ related strategies such that at most one fails" imply a single strategy that fails with probability $\le 1/n$?

In a generalized "prisoners/hats/boxes"-style logic puzzle, suppose we have the following: a set $W$ (of "possible world configurations") a set $S$ (of "possible individual ...
Karl's user avatar
  • 11.5k
1 vote
0 answers
44 views

Gauss-Markov characterising property

I am trying to solve exercise no. 1.13 page 86 of 'Continuous time Martingales & Brownian Motion', by Revouz-Yor. It says that a stochastic process $X_t$ is a Guassian process (i.e. the marginal ...
Tralfamadorian26's user avatar
0 votes
1 answer
77 views

Show that $\frac{1}{n} \sum \limits_{k=1}^{n} X_{k}-\frac{1}{n} \sum \limits_{k=1}^{n} \mathbb{E} X_{k} \stackrel{\text { f.s. }}{\longrightarrow} 0$

Let $ \left(X_{n}\right)_{n \in \mathbb{N}} \subset \mathcal{L}^{2} $ be a sequence of independent random variables. Furthermore, let $ \alpha<1 $ and $ c \geq 0 $ be such that $ \sum \limits_{i=1}^...
clementine1001's user avatar
0 votes
2 answers
56 views

equality of conditional expectation

Let $X_1,...,X_k$ be independent exponentially distributed with parameter $\lambda$ random variables. Is this equality holds? $$E[e^{-X_1-...-X_k}|X_1+...+X_k]=e^{-X_1-...-X_k}$$ My textbook says it ...
romperextremeabuser's user avatar
0 votes
1 answer
181 views

100 People Throwing D100

If one person throws 100 times a fair dice with 100 corners (numbered from 1 to 100), then their probability to roll 100 at least once in any of those times equals: $$P(X=100)=1-\left(\frac{99}{100}\...
E.Z's user avatar
  • 273
2 votes
1 answer
81 views

Show that $ \frac{Y_{1}+\cdots+Y_{n}}{n} \stackrel{\mathrm{P}}{\longrightarrow} \mu \text { as } n \rightarrow \infty $

Let $Y_{1}, Y_{2}, \ldots$ be a sequence of identically distributed random variables with $\mathbb{E} Y_{1}=\mu$ and $\operatorname{Var} Y_{1}=\sigma^{2}<\infty$. Furthermore, let $Y_{j}$ be ...
clementine1001's user avatar
2 votes
0 answers
66 views

Essential Supremum over a set of random variables attaining maximum

I think the following result is true, but I cannot find any source online, so I post it here for confirmation: Fix a probability space $(\Omega,\mathcal{F},\mathbb{P})$, an arbitrary index set $\...
LonerSeekDefeat's user avatar
0 votes
1 answer
115 views

residual waiting time and spent time

Does the expected value of the residual time (the amount of time one has to wait) equal the expected value of the spent time (the amount of time since the last arrival)? I know that the expected value ...
APerson's user avatar
  • 85
0 votes
0 answers
33 views

Existence of a Unif(0,1) random variable for counter-monotonicity

We know that for a pair of counter-monotonic random variables $X$ and $Y$, it holds that \begin{equation} (X,Y)\sim\big(q_X(U),q_Y(1-U)\big) \end{equation} for $U\sim\text{Unif}(0,1)$ and $q_X,q_Y$ ...
Seraph Yang's user avatar
1 vote
0 answers
62 views

Question about showing stopping time

I'm having a trouble about how to show $T$ is a stopping time. For instance, suppose $X$ is a simple random walk on $I = \mathbb{Z}/k\mathbb{Z}$, and $T$ is the first time $n$ such that $X_0, \dots, ...
Uther's user avatar
  • 21
2 votes
2 answers
87 views

Is the normal distribution where the parameters themselves are normal random variables still normal?

We denote by $\mathcal{N}(\mu, \sigma^2)$ the normal distribution where $\mu\in\mathbb{R}$ is the mean and $\sigma\in\mathbb{R}$ is the scale parameter. Assume that the parameters $\mu,\sigma$ are ...
Levent's user avatar
  • 4,814
1 vote
1 answer
98 views

random walk, expected value of visiting point before back to origin

Consider standard random walk on integer line. (probabilities $\frac{1}{2}$ to go left or right, start at $0$). I am reading probability and random processes book and I came across following theorem: ...
romperextremeabuser's user avatar
1 vote
0 answers
62 views

derivation of exponential distribution's expectation using poisson

I know that for a Pois$(\lambda)$ distribution, the expectation is $\lambda$. I also know its relation to the exponential distribution, and how you can derive the PMF of the exponential through the ...
APerson's user avatar
  • 85
0 votes
1 answer
51 views

reccurence relation, particular solution does not work

I have linear nonhomogeneous reccurence equation: $$pa_n-a_{n=1}+(1-p)a_{n-2}+1=0$$ I solve homogeneous equation first and I get: $a_n=A+B(\frac{1-p}{p})^n$ Now I want to guess particular solution, so ...
romperextremeabuser's user avatar

1
49 50
51
52 53
891